Làm thế nào tốn kém có thể phá hủy tất cả các con đường dài trong DAG?


14

Chúng tôi xem xét các DAG (đồ thị chu kỳ có hướng) với một nút nguồn s và một nút mục tiêu t ; các cạnh song song nối cùng một cặp đỉnh được cho phép. Một k - cut là một tập hợp các cạnh mà việc loại bỏ sẽ phá hủy tất cả đường dẫn ss - t dài hơn k ; ngắn s - t đường dẫn cũng như dài đường dẫn "bên trong" (những người không giữa st ) có thể sống sót!

Câu hỏi: Có đủ để loại bỏ tối đa là khoảng 1/k phần cạnh từ một DAG để tiêu diệt tất cả s - t con đường dài hơn k ?

Nghĩa là, nếu e(G) biểu thị tổng số cạnh trong G , thì mỗi DAG G có một k -cut với nhiều nhất là về các cạnh e(G)/k không? Hai ví dụ:

  1. Nếu tất cả các s - t con đường có chiều dài >k , sau đó một k -cut với e(G)/k cạnh tồn tại. Điều này được giữ bởi vì sau đó phải có k disjoint k -cut: chỉ lớp các nút của G theo khoảng cách của chúng từ nút nguồn s .
  2. Nếu G=Tn là một giải đấu bắc cầu (một DAG hoàn chỉnh), thì cũng là một k -cut với tồn tại: sửa một thứ tự tôpôcủa các nút, chia các nút thành các khoảngkliên tiếp có độ dàin/kvà loại bỏ tất cả các cạnh nối với các nút có cùng khoảng; điều này sẽ phá hủy tất cảcácđường dẫns-tdài hơnk. k(n/k2)e(G)/kkn/kstk

Lưu ý 1: Một nỗ lực ngây thơ để đưa ra một câu trả lời tích cực (mà tôi cũng đã thử trước tiên) sẽ là cố gắng thể hiện rằng mọi DAG phải có khoảng rời rạck -cuts. Thật không may, Ví dụ 2 cho thấy rằng nỗ lực này nặng có thể thất bại: thông qua một cuộc tranh luận thoải mái, David Eppstein đãcho thấyrằng, đối với k vềkk , đồ thịn không thể có nhiều hơn bốnrời nhau k -cuts! Tn k

Ghi chú 2: Điều quan trọng là một -cut chỉ cần tiêu diệt tất cả dài s - t con đường, và không nhất thiết tất cả các đường dẫn dài. Cụ thể, tồn tại 1 DAG trong đó mọi k -cut "thuần" (tránh sự cố cạnh cho s hoặc tkstkst ) phải chứa gần như tất cả các cạnh. Vì vậy, câu hỏi của tôi thực sự là: khả năng loại bỏ các sự cố cạnh với hoặc t có thể làm giảm đáng kể kích thước của một k -cut không? Rất có thể, câu trả lời là tiêu cực, nhưng tôi chưa thể tìm thấy một ví dụ nào. stk

Động lực: Câu hỏi của tôi được thúc đẩy bằng cách chứng minh giới hạn thấp hơn cho các mạng chuyển mạch và chỉnh lưu đơn điệu. Một mạng như vậy chỉ là một DAG, một số cạnh có nhãn được kiểm tra "là ?" (không có bài kiểm tra x i = 0 ). Các kích thước của một mạng lưới là số cạnh dán nhãn. Một vectơ đầu vào được chấp nhận, nếu có một đường dẫn s - t thì tất cả các phép thử của chúng đều phù hợp với vectơ này. Markov đã chứng minh rằng, nếu một hàm boolean đơn điệu w là cần thiết. Một câu trả lời tích cực cho câu hỏi của tôi sẽ ngụ ý rằng mạng lưới kích thước khoảng k xi=1xi=0st không có minterms ngắn hơn l và không có maxterms ngắn hơn w , thì kích thước lflwlw là cần thiết, nếu ít nhấtcác biến w k phải được đặt thành 0 để phá hủy tất cả các minterms dài hơn k .kwkwk0k


1 Việc xây dựng được đưa ra trong bài báo này. Lấy một cây nhị phân hoàn chỉnh của log sâu n . Loại bỏ tất cả các cạnh. Đối với mỗi nút bên trong v , vẽ một cạnh cho v từ mỗi lá của cây con bên trái của T v và một cạnh từ v đến mỗi lá của cây con bên phải của T v . Do đó, cứ hai lá T được nối với nhau bằng một đường dẫn có độ dài 2 trong DAG. Các DAG chính nó có ~ n nút và ~ n log n cạnh, nhưng Ω ( nTlognvvTvvTvT2nnlogn cạnh phải được loại bỏ để tiêu diệt tất cả những con đường dài hơnΩ(nlogn) .n


Dòng chảy giới hạn và cắt giảm có liên quan chặt chẽ với các câu hỏi bạn yêu cầu. Tôi khuyên bạn nên nhìn vào luận án của Baier. ftp.math.tu-berlin.de/pub/Preprints/combi/ory
Chandra Chekuri

@Chandra Chekuri: cảm ơn vì liên kết thú vị. Luận án nói nhiều hơn về định lý của Miller về các đường đi / sai sót ngắn . Liên quan đến Menger cho các đường dẫn dài , tôi tìm thấy bài báo này : kích thước tối thiểu của k-cut nhiều nhất gấp khoảng k lần số lượng tối đa của các đường dẫn tách rời dài. Nhưng điều này dường như cũng không giúp được gì.
Stasys

Xin lỗi, tôi đã hiểu nhầm câu hỏi. Cảm ơn các tài liệu tham khảo khác.
Chandra Chekuri

Câu trả lời:


8

[Tự trả lời; đây là phiên bản rút gọn, bản cũ có thể tìm thấy ở đây ]

Chúng tôi nhận ra với Georg Schnitger rằng câu trả lời cho câu hỏi của tôi rất tiêu cực : có DAG (thậm chí ở mức độ không đổi), trong đó mọi -cut phải có một phần không đổi của tất cả các cạnh, không chỉ là một phần khoảng 1 / k , như trong câu hỏi của tôi. (Một kết quả yếu hơn một chút có thể cần đến phân số 1 / log k bằng cách sử dụng một cấu trúc đơn giản hơn nhiều được đề cập trong phần chú thích ở trên. Một bài viết nhanh ở đây ) k1/k1/logk

Cụ thể, trong bài báo "Giảm độ sâu và lưới" , Georg đã xây dựng một chuỗi các đồ thị chu kỳ có hướng có độ lớn tối đa không đổi d trên các nút n = m 2 m với thuộc tính sau:Hndn=m2m

  • Đối với mỗi hằng số có một hằng số c > 0 như vậy mà, nếu có tập hợp con của ít nhất c n nút được lấy ra từ H n , đồ thị còn lại chứa một con đường có độ dài ít nhất 2 ε m . 0ϵ<1c>0cnHn2ϵm

Bây giờ lấy hai nút mới t và vẽ một cạnh từ s đến mọi nút của H n và một cạnh từ mọi nút của H n đến t . Đồ thị kết quả G n vẫn có tối đa 2 n + d n = O ( n ) .stsHnHntGn2n+dn=O(n)

Đối với mỗi hằng số , có một hằng số c ' > 0 như vậy mà, nếu có tập hợp con của tối đa là c ' n cạnh được tách ra từ G n , đồ thị còn lại chứa một s - t con đường với 2 ε m hoặc nhiều cạnh hơn. 0ϵ<1c>0cnGnst2ϵm

Chứng minh: Gọi các nút của nút bên trong của G n . Loại bỏ bất kỳ tập hợp con nào của hầu hết các cạnh c n khỏi G n , trong đó c = c / 2 . Sau đó, loại bỏ một nút bên trong nếu nó là sự cố với một cạnh bị loại bỏ. Lưu ý rằng tối đa 2 c n = c n các nút bên trong sẽ bị xóa. Không có sự cố cạnh nào cho các nút còn sống sót được gỡ bỏ. Cụ thể, mỗi nút bên trong còn tồn tại vẫn được kết nối với cả hai nút stHn GncnGnc=c/22cn=cnst. Theo tính chất trên của , vẫn phải có một đường dẫn có chiều dài 2 ϵ m bao gồm toàn bộ các nút bên trong còn sót lại. Vì các điểm cuối của mỗi đường dẫn này còn tồn tại, mỗi đường dẫn có thể được mở rộng thành đường dẫn s - t trong G n . QEDHn2ϵmstGn

Một hậu quả thật đáng buồn: không tồn tại bất kỳ sự tương tự nào của bổ đề của Markov đối với các hàm có nhiều minterms ngắn , mặc dù tập hợp các minterms dài có cấu trúc "phức tạp": không thể chứng minh được giới hạn siêu tuyến tính trên kích thước mạng đối số "chiều dài lần chiều rộng" này.

PS này "lần chiều dài chiều rộng" đối số (khi tất cả - t con đường đủ dài) đã sớm được sử dụng bởi Moore và Shannon (1956). Sự khác biệt duy nhất là chúng không cho phép chỉnh lưu (các cạnh không được gắn nhãn). Vì vậy, trên thực tế, đây là một "đối số Moore-Shannon-Markov".st

Khi sử dụng trang web của chúng tôi, bạn xác nhận rằng bạn đã đọc và hiểu Chính sách cookieChính sách bảo mật của chúng tôi.
Licensed under cc by-sa 3.0 with attribution required.